User:Btilm305/testproblem

< User:Btilm305
Revision as of 15:48, 3 June 2011 by Btilm305 (talk | contribs) (see what happens with a longer problem)
(diff) ← Older revision | Latest revision (diff) | Newer revision → (diff)

Solve for $x$. $x+12=15$. Bob and Janna blah blah blah go to the store. Let me copy another problem... Let $x_1, x_2, ... , x_6$ be non-negative real numbers such that $x_1 +x_2 +x_3 +x_4 +x_5 +x_6 =1$, and $x_1 x_3 x_5 +x_2 x_4 x_6 \ge {\scriptstyle\frac{1}{540}}$. Let $p$ and $q$ be positive relatively prime integers such that $\frac{p}{q}$ is the maximum possible value of $x_1 x_2 x_3 + x_2 x_3 x_4 +x_3 x_4 x_5 +x_4 x_5 x_6 +x_5 x_6 x_1 +x_6 x_1 x_2$. Find $p+q$.